PT44.S2.Q13 - the solution to any

Accounts PlayableAccounts Playable Live Sage
edited February 2016 in Logical Reasoning 3107 karma
This is a hard SA question, and I don't see what I am missing. How is answer choice A a sufficient assumption?

My diagram:
(Solution to environmental problem not caused by the government)--->(Major change in consumer habits)--->(Economically Enticing)
Therefore, (Not Economically enticing)--->(Few serious ecological problems solved).

What I am looking for: This is a pretty simply A to B to C argument, and the conclusion as a Not C in it. To link up the chain, say (Not solution to environmental problem not caused by the government)--->(Few serious ecological problems solved).

Answers B-E are way wrong, but I don't see how answer A paraphrases the sufficient assumption at all.

Link: https://7sage.com/lsat_explanations/lsat-44-section-2-question-13/

Comments

  • DumbHollywoodActorDumbHollywoodActor Alum Inactive ⭐
    7468 karma
    I think the first sentence is an embedded conditional. I translated that first sentence of the stimulus to “If there’s a solution to any environmental problem and it’s not caused by government mismanagement, then it requires major changes in consumer habits.”

    Contraposed (using De Morgan’s Law); "If there’s no major changes, then either there’s no solution to the problem or it’s caused by government mismanagement.”

    I’ll be the first to admit it. I totally reverse engineered that one. But I can’t think of any other way to make A a sufficient assumption.
  • Elle2015Elle2015 Alum Member
    198 karma
    If all serious problems are caused by the government, the conclusion is wrecked because their solutions might not need to be economically enticing. Perhaps they could be fixed by electing new officials.

    I think A includes that idea. I'm not sure how you're trying to link up the chain.
  • DumbHollywoodActorDumbHollywoodActor Alum Inactive ⭐
    7468 karma
    @Elle2015 If this were a necessary assumption question, then I think you’d be on to something, employing the negation test. But this is a sufficient assumption question. So we need an answer that will make the argument valid.
  • DumbHollywoodActorDumbHollywoodActor Alum Inactive ⭐
    7468 karma
    Here’s my answer explanation to the question for what it’s worth. Please feel free to offer criticism or clarification:
    This is an SA question:

    Premise: A solution that is not the result of government mismanagement requires major changes in consumer habits, Major changes in consumer habits require changes that are economically enticing (/GM→MC→EE)

    Conclusion: If solutions aren’t made economically enticing, then few serious ecological problems will be solved. (/EE→FES)

    Analysis: The structure is
    A→B→C
    ———————
    /C→D

    We need to fill the gap. The sufficient condition of the conclusion triggers the the premise chain by denying the its necessary condition. The gap could be filled by either /B→D or /A→D. The D represents the new concept mentioned in the conclusion (few ecological problems), and we’ll need to make that a necessary condition to either the negation of B [no major changes (/MC)] or the negation of A [the result of government mismanagement (GM)]

    Answer A fits the bill. All of the other answers are out of the scope of the argument’s gap.

  • runiggyrunruniggyrun Alum Inactive Sage Inactive ⭐
    2481 karma
    The first statement splits the world of environmental problems in two: the ones caused by government mismanagement and the rest of them. The whole remainder of the argument deals with the subworld of problems that are not the result of government, and if the conclusion were phrased "few serious ecological problems not caused by government mismanagement will be solved unless the solutions are made economically enticing", then this would be a valid argument. But it doesn't - the conclusion refers to the whole world of environmental problems, and therefore the gap to be bridged to make the argument valid is the gap between these two worlds: the complete world of the conclusion, and the partial world of the argument.
    Answer A bridges that gap. If only few problems are a result of government management, then the two worlds overlap almost perfectly. Only a few problems from the whole world will be outside of the world discussed by the argument. Since the conclusion doesn't say "No problems will be solved, but "few" problems will be solved" an almost perfect overlap is good enough to guarantee the conclusion.
  • DumbHollywoodActorDumbHollywoodActor Alum Inactive ⭐
    7468 karma
    @runiggyrun said:
    The first statement splits the world of environmental problems in two: the ones caused by government mismanagement and the rest of them.
    FTW. Nicely done!
  • Elle2015Elle2015 Alum Member
    198 karma
    @DumbHollywoodActor Sorry. I didn't mean to imply that one should use the negation test, but just that something needed to account for the set of problems that were govt. created. Luckily @runiggyrun explained it well.
Sign In or Register to comment.